All posts

New post

217 posts in the last 30 days

User Avatar

Last comment friday, nov 06 2015

Games taking me ~12-13 minutes

I have been working on my LG for the last few days, in preparation for the December test. I am now getting to the point where I can get all the questions right on a standard linear game, but it's taking me a minimum of 12 to 13 minutes, and sometimes as much as 15 minutes per game.

I have a hard time seeing how I can increase my speed. For me, the games just require a lot of thought and that takes some time.

Anyone else faced this issue? Right now, 8 minutes per game seems almost impossible. Is it just a matter of practice, and that taking a lot of tests will increase speed?

0
User Avatar

Last comment thursday, nov 05 2015

UCLA Study Partner/Group

Hi y'all, I'm studying for the upcoming Dec LSAT and am currently scoring in the low 160's. I'm interested in working with someone(s) to do PT'S/BR/Drill etc. Shoot my inbox and let's set something up to crush the LSAT. It's almost time!

0

Just curious if being significantly older than most students applying to law school (lets say hypothetically speaking 20+ years) would qualify you as being URM? Not that I would actually know anyone who was of course :)

0

Absolutely clueless on this one. I've probably watched the video 5 times, and I just don't see how B weakens the argument that is in the passage. Also, can someone look at my reasoning for A?

Win democratic elections not fully subsidized--->poor candidates supported by rich. As a result, it could be true that the poor candidates will compromise their positions to win the rich guy's support. However, the proportion of rich people in all of the political parties is the same as their proportion in the population. Thus, the belief that it could be true that poor candidates will compromise their position to win the rich people's support is wrong.

What I am looking for: The premise doesn't support the conclusion at all since we don't know whether being proportionally equal nullifies the pressure to conform to the rich guy's opinion. For example, say that rich people are 1% of the population in all political parties; they must then be 1% of the total population. It might be reasonable to say that the rich don't have that big of an influence on policy. Now, what if the proportion was 99%? The rich might have a huge say in policy! Thus, the premise could go both ways in either providing support for the conclusion or not.

Answer A: This is what I chose, but I still am very unsure why it is wrong. I think the argument does fail to address this answer choice. Maybe it is wrong because of the word "primary?" I'm not so sure though since we are usually supposed to accept the answer choices as true. If it is the case that the "primary" function of a party still may not negate the influence of wealth, then doesn't this paraphrase the flaw very well? To me, this is hinting at the idea that the proportion of the wealthy might be so high (or so influential) in political parties that it doesn't matter which one the poor candidate choose, they will have to conform to the wealthy point of view's party.

Say that the word "primary" is the reason why this is wrong (which I am not really sure why it would be), what if this answer choice said "a function of political parties...?" Would it be correct then?

Answer B: Like I said earlier, I just don't see how this weakens the argument. I do think it weakens an argument, just not the one in the passage. Here is my breakdown of this answer choice: say the poor candidate believes, "Every person who takes the LSAT should get a 180 and full ride to Yale" but the Democrats and Republicans both think "absolutely not" (the positions of the parties is way less varied than the position of the candidate). Then sure, joining a political party would compromise the poor candidate's views. But, that isn't the argument in the passage. The argument is that the "possibility of a poor candidate compromising his views to win the support OF THE RICH [not the political party] is not true." How are these two the same argument? In other words, if this is true, isn't the LSAT equivocating between the views of the "political parties" and the views of the "wealthy patrons?"

Answer C: We don't care about government subsidized elections.

Answer D: We don't care about wealthy candidates.

Answer E: We don't care about other flaws.

0

What lessons are the best to review for mapping out logic? Perhaps maybe a couple rule of thumbs to always go back to would be great! I use to do very well on SA questions and now I see myself getting more than half of them wrong, I know these type of questions usually play leave on mapping out the logic.

0
User Avatar

Last comment thursday, nov 05 2015

Study Plan

I'm trying to figure out the best way to make use of the 7Sage Core curriculum, the LSAT Trainer, and a bunch of grouped LR questions based on question-type (going up to PT 30) which I'm using before I go into the whole PT/ BR phase in preparation for the June 2016 test. If you've used both 7Sage and the Trainer how did you approach your studies? Is it generally advisable to go through the course and then read the Trainer or vice-versa? Would you make use of the grouped question-types at the end of each lecture on that specific type or use them later for review? I'm considering starting the whole PT/BR regimen at the end of January at the latest which would give me at least a solid 4 months before the test. I really need to structure my studies so any advice/ comments going forward would be great.

0
User Avatar

Last comment thursday, nov 05 2015

Logging PT errors

Just curious if anyone keeps a spreadsheet or anything of the questions they get wrong on the PTs? Or do you all just track it in LSAT Analytics? If you do log the questions you are are getting wrong in a spreadsheet, do you actually write out the questions, answer choices etc. and make notes of why you got it wrong? Or do you just write the test number section, & question & type of question. Just trying to figure out if it would be helpful for me to do this or if it would just be wasting valuable time I could be spending more productively. I'm already doing a BR. Although I think I'm going to change the way I have been doing that. I am going to start doing an untimed BR of the complete exam instead of just reviewing the ones I circled in addition to the ones I actually got wrong. Any tips or suggestions would be greatly appreciated.

0

This question is difficult because once you spot the flaw, it is hard to put it into words, which is why I missed it. I couldn't figure out how any of the answer choices paraphrased the flaw, so I had to pick an answer and move on. I don't really see how C is the flaw and how A is worse than C.

Bike riders don't follow the rules of the road, and this is a causal factor in 25% of traffic accidents involving bikes. The lack of bike saftey equipment is also a causal factor in 25% of those accidents. Thus, bikes are partly responsible for more than half of the traffic accidents involving bikes.

What I am looking for: I think the flaw is a math error. The conclusion says that 50+%, but we are given information about a causal factor being 25% and of those accidents a causal factor is 25%. Instead of additive, the relationship should be multiplicative. The conclusion should only talk about the percentage of bike accidents that included inadequate bike saftey equipment.

Answer A: This was the answer I chose, and I don't see how this doesn't adequately point out the flaw. Sure, you need to make an assumption that motorists are a factor in traffic accidents, but how is that not a reasonable assumption that the author overlooked? Additionally, since we conclude that 50+%, this is implying that less than 50% ("less than half") have some other cause. But, since we can't conclude anything about 50+%, this presumption is not justified. I don't see what is wrong with this one.

Answer B: No. We are to take the causal premises as truth.

Answer C: How is this the correct answer choice? Doesn't the conclusion say "at least partially responsible?" Thus, the argument DOES consider the possibility that more than one factor may contribute to a given accident? Additionally, the argument isn't talking about "all/given accidents;" it is limited to accidents involving bikes. How can the flaw be about "given" accidents?

Answer D: We don't need a source.

Answer E: Who cares about the severity of injury?

0
User Avatar

Last comment thursday, nov 05 2015

PT 70 S1 Q23

I've looked at about 3 different courses with explanations for this question and none of them really make an sense to me... I still see answer choice (B) as an attractive answer choice... Can someone please explain why all the answers are incorrect and perhaps a better way to approach PSA questions for next time? GREATLY APPRECIATED!

0
User Avatar

Last comment thursday, nov 05 2015

57.2.2 Taxi drivers

I am just not seeing this one. I understand the gist of what answer B is saying, but I don't understand how the assumptions you need to make in the other answer choices are necessarily better than the assumption needed for B to be the correct answer.

The question is a must be false question, so 4 answer choices could be true.

Taxi drivers earn income based on the fares they get. The decide their working hours by setting an income target, and they stop working when they hit the target. Thus, they typically work fewer hours on busy days than on slow days.

What I am looking for: Essentially, the drivers at the start of the day say, "I will stop once I make $100." Once they hit that, they stop working. How does this suggest they stop working earlier on busy days? Couldn't they set their target higher on those days (assuming busy days are predictable)?

Answer A: Ya, I guess the argument supports this idea. The driver himself sets the target, so unless the driver isn't rational, he sets the target based on his needs. This could be true.

Answer B: I still really don't like this answer choice. It is true that if you set the target for $100 and you work 1 hour on busy days, the EHW is $100/hour. On slow days, say you work 10 hours, your EHW would be $10/hour. Thus, the passage seems to suggest that the opposite of this answer choice is true: you work less when your EHW is high (on busy days). But, what if they change their income targets on busy days to even out their EHW? You have to assume the drivers don't do this. Given that answer choices C-E also need assumptions, how do we value this assumption over the others? For this reason couldn't this answer choice also be true? I just don't see how we have enough information to say it must be false.

Answer C: It's true that the drivers get to set their own schedule, so I guess it could be true they accept a lower wage.

Answer D: Same as D. You have to assume that the drivers take into account their standard of living when they set their target. This could be true.

Answer E: We don't know anything about people with fixed hourly wages, so any comparison could be true.

0

Hey 7Sagers, had someone email me with a question and I thought you guys could help out! Here it is:

I have a question or comment regarding LSAT 46, game 3, section 4, question 12. The game ask for the order.

In accordance to the rules P=3 and L and N are before J. The rules don't state that L or N is immediately next to J. Thereby the explained set up is:

L N P J O or L/O N P L/O J. As with the rules, I agree and can understand this except for N = 2. With L = 4, that places L immediately next to J for LJ.

That is my problem. If LJ then why not LN. For set up with J=5; L = 1, 2, 4; N =1,2, 4, and O = 1,2,4,5

For order choices question 12

L N P J O O N P L J N L P J O

O N P L J N O P L J L O P N J and so forth.

The answer choice

L N P O J - Answer A deemed correct answer choice.

L O P N J - Answer B deemed incorrect answer choice. However, both L and N are before J with P =3

N L P J O - Answer D deemed incorrect answer choice. However, both L and N are before J with P = 3

Please explain. Hail and rain is not a factor to answer this question. In addition, the rules and the scenario do not include N = 2.

Thank you.

0
User Avatar

Thursday, Nov 05 2015

Advice

Hey all, Al here.

Throughout my long (and I mean LONG) journey with the LSAT, I've come across this specific issue more times than I can count! I'd like to give my own personal thoughts on this topic.

Fellow LSAT students (non-7sage included) have reached out to me time and time again about this recurring topic: advice. I think it's safe to say that many (if not most) people think advice is inherently good. It is after all rooted in the premise that advice helps to inform and guides those that are lacking in certain information that can help others in the long run. And in many respects, the right advice can provide an enormous positive impact on the person receiving it.

But what's rarely talked about are the devastating effects of bad advice. Bad advice can lead us to do things that we normally wouldn't consider and the negating effect can be as substantial as any good advice. Not only does it deviate a person from one's initial trajectory, but it can undo a lot of the progress they have made on their own.

But what really is the distinction between good and bad advice? Here's what I think are the major distinctions:

1. Advice is inherently subjective. What may be great advice for one person maybe horrible advice for another.

2. Good advice has specificity, whereas bad advice is openly vague. The LSAT is a great example of this. So many of the nuances embedded within concepts can be very hard to absorb and the advice provided has to not only be logically congruent within the parameters of the LSAT...it also has to make absolute sense to the person receiving it. I've seen many people (including myself many times) THINK they have an understanding of a certain tidbit of advice but really have no understanding at the core of it.

3. Good advice more often than not takes time, whereas bad advice is usually impulsive. I've noticed this for myself when I've been asked for advice. When I have the time to really think about a problem, I more often than not have something worth telling the person asking for advice.

I have been the benefactor of both great and bad advice. All three of these tenets that I've mentioned applied to me. Yes, it does suck when we get bad advice. But the onus cannot be entirely on the person providing the advice...the onus is on the one receiving it. At the end of the day, it's all about personal responsibility and knowing what truly works and what doesn't work for you.

Just food for thought.

3

Hello 7sagers,

I just signed up a while ago and thought I'd give this forum a shot. I was wondering on logic games, when do you create a table between the game pieces and the base set like in the Feb 1993, Logic Game #4? Sometimes I'll create a table when it is not needed. Thanks for all the help!

0
User Avatar

Last comment wednesday, nov 04 2015

57.2.25 The law of the city

EDIT: I had the wrong question in the title

I have a huge bone to pick with this one. I don't see how any of the answer choices "must be true."

Here is the law:

Greater than $100 AND made by nonresident AND nonresident isn't former resident--->Register

The campaign complied with the law (so it complied with the contrapositive as well). The campaign accepted contributions only from residents and former residents.

What I am looking for: I thought this was a pretty weird passage since the final clause only denies the sufficient condition, which tells us nothing about if those contributions needed to be registered. Thus, I thought an answer choice was going to specifically reaffirm this premise.

Answer A: This is what I chose, even though I was confident it was wrong. I chose it since I thought all of the answer choices didn't work. This answer is incorrect because the dollar amount of the people that donated to the campaign is irrelevant. The nonresident in this answer choice MUST have been former resident, and if this were what this had said, it would have been correct. However, the dollar amount could have been anything since our conditional rule is irrelevant.

Answer B: This could be true, but it doesn't have to be true. We only know one sufficient condition for registration; there could be multiple sufficient conditions for registration.

Answer C: This is the correct answer??? How must this be true? We only know one rule/sufficient condition for registration. Why can't there be other rules? The passage never indicates that the rule given is the only rule. What if all contributions from residents and former residents must be registered? How this is even close to being a correct answer is beyond my understanding.

Answer D: Again, this doesn't have to be true. We know nothing about donations by the residents and former residents. They each could have given the campaign $1 or something.

Answer E: Again, we only have one sufficiency condition for registration, so this could be true.

0
User Avatar

Last comment wednesday, nov 04 2015

LR

What is the best method to program yourself to look for incorrect answers, as opposed to correct answers, during the LR section?

I was told that this is the most effective approach to do very well within this section. What do you think?

0
User Avatar

Last comment wednesday, nov 04 2015

Outside Reading Suggestions

Someone last time recommended "The Great Dissent" by Thomas Healy and I absolutely loved the book.

Figured it would be good to start another thread of book/article recommendations that can help improve reading skills + make you more familiar with LSAT subject matter.I struggle with Humanities passages so if anyone has any recommendations...

I mentioned last time that Bill Bryson's " A Short History of Nearly Everything" is a great read: will help you become familiar with a lot of the topics that are tested on the science passages and is also interesting.

0
User Avatar

Last comment wednesday, nov 04 2015

Clean copy BR process

Just to clarify when you LSAT PT rockstars do your clean copy BR, are you just BRing the ones your circled during your PT or are you going back through every question untimed? Thanks in advance.

1

I know there was a discussion thread going on about the 180 watch or just a test watch in general... But, I wanted to get a confirmation. Is it okay to bring multiple watches?

I was going to order the 180 watch.... but, I guess they ran out of inventory or something because the watch is on back order.. So, I am wondering whether anyone has brought 3+ watches, set them all for 35 minutes, and used them for each different section?

I would really appreciate all the answers! Thank you!

0
User Avatar

Last comment wednesday, nov 04 2015

Someone please help!

I just took my first PT in months to prepare for the December LSAT. I finished the course a couple days ago and I scored a 151, and after BR 162. I did miss a significant amount of questions in each section averaging about 7 in each. I'm not sure where to go from here... Should I keep PTing until the exam or....? PLEASE HELP!

Admin note: Please don't post thread titles in all caps!

0

Confirm action

Are you sure?